2010014902

Parte: 
Project ID: 
2010014902
Source Problem: 
Accepted: 
0
Clonable: 
1
Easy: 
0
El conjunto de soluciones de la inecuación \( \mathrm{tg}\, x > -\frac{\sqrt3}3 \) para \( x\in\mathbb{R} \) es:
\( \bigcup\limits_{k\in\mathbb{Z}}\left(-\frac{\pi}6+k\pi;\ \frac{\pi}2+k\pi\right) \)
\( \bigcup\limits_{k\in\mathbb{Z}}\left(-\frac{\pi}3+k\pi;\ \frac{\pi}2+k\pi\right) \)
\( \bigcup\limits_{k\in\mathbb{Z}}\left(-\frac{\pi}6+k\pi;\ \frac{\pi}6+k\pi\right)\)
\( \bigcup\limits_{k\in\mathbb{Z}}\left(-\frac{\pi}6+k\pi;\ \pi+k\pi\right) \)